CLAT UG 2022 Solved Paper | CLAT UG Entrance Solved Papers Pdf

Find CLAT UG 2022 Solved Paper on Legal Bites. Owing to the increasing competition day by day, getting into your dream college is not that easy. Practice the CLAT UG past year paper to reflect on your preparation and increase your knowledge with the correct information. Practice makes a man perfect and thus, solving the past year paper will provide you with an edge over your competitors. Click Here for Online Mock Tests and Solve Live.

This will allow you to grasp different concepts and assist you in developing a framework and strategy of preparation. The scores will further provide you with an analysis of your weaknesses and strengths. Attempting the paper will familiarize you with the pattern, structure and difficulty of the paper and help you ace your exams.

Find the solved CLAT UG paper below.

CLAT UG QUESTION PAPER 2022
No. of questions- 150

English Language

I. Public speaking is a powerful real-life skill. Over the centuries, impressive speeches made by people from various walks of life have helped to change hearts, minds and shape the world as we see it today. Speeches that are delivered with intense emotions and conviction can infuse compassion and forgiveness; elevate levels of hatred and destruction; break or unite nations.

On October 5, in 1877 in the mountains of Montana Territory, when Chief Joseph surrendered to General Nelson A. Miles, the former gave a Surrender Speech. The speech included these words: “It is cold, and we have no blankets; the little children are freezing to death. I want time to look for my children and see how many of them I can find. Maybe I shall find them among the dead. Hear me, my Chiefs! I am tired; my heart is sick and sad. From where the sun now stands I will fight no more forever.”

The heart-wrenching speech bared the grief and misery of the speaker, and those subjected to overwhelming hardships.

During World War II, the speech We Shall Fight on the Beaches delivered by Winston Churchill on June 4, 1940 is considered a high-powered speech that strengthened the determination of those present in the House of Commons. In the speech, he said, “Even though large tracts of Europe and many old and famous States have fallen or may fall into the grip of the Gestapo and all the odious apparatus of Nazi rule, we shall not flag or fail. We shall go on to the end, we shall fight in France, we shall fight on the seas and oceans, we shall fight with growing confidence and growing strength in the air, we shall defend our island, whatever the cost may be, we shall fight on the beaches, we shall fight on the landing grounds, we shall fight in the fields and in the streets, we shall fight in the hills;”

In 1950, William Faulkner was honoured with a Nobel Prize for his significant contributions to the American novel. This was the time when the Soviet Union had found the possible implications of the use of the atomic bomb, and people had begun to live in the fear of annihilation. In his Nobel Prize Acceptance Speech, Faulkner urged writers of various genres to think and write beyond the fear of destruction, and instead write materials that would lift the human spirit. The powerful message included: “I believe that man will not merely endure: he will prevail. He is immortal, not because he alone among creatures has an inexhaustible voice, but because he has a soul, a spirit capable of compassion and sacrifice and endurance. The poet’s, the writer’s, duty is to write about these things. It is his privilege to help man endure by lifting his heart, by reminding him of the courage and honor and hope and pride and compassion and pity and sacrifice which have been the glories of his past. The poet’s voice need not merely be the record of man, it can be one of the props, the pillars to help him endure and prevail.”

1. Undoubtedly, effective speeches have a long-lasting impact on the minds of the listeners, and they elevate the levels of awareness or actions the speaker intends to raise or catalyze.

1. The main idea of the passage is that

(A) All leaders should be accomplished public speakers.

(B) An impactful speech can convey a strong message to the listeners.

(C) A speech should sound pleasing to the ears of the listeners.

(D) Public speakers should be bold and argumentative.

2. The tone of the Surrender Speech is

(A) Satiric

(B) Optimistic

(C) Poignant

(D) Narcissistic

3. It is evident that through his speech, Churchill wished to his countrymen .

(A) Inform, about the challenges that arise in a war-torn country.

(B) Warn, against the futility of war.

(C) Remind, how their endeavours to fight against the Nazi rule had failed miserably.

(D) Reassure, that they would combat fiercely against their enemy under all circumstances.

4. Which one of the following is the least likely to be used to describe Churchill?

(A) Resolute

(B) Undaunted

(C) Complacent

(D) Unwavering

5. In the sentence: ‘The poet’s voice need not merely be the record of man, it can be one of the props, the pillars to help him endure and prevail’ Faulkner has used to convey the power of a poet’s writings.

(A) A metaphor

(B) A simile

(C) An onomatopoeia

(D) A transferred epithet

II. As a six-year-old child-beggar, Saroo slept off in a stationary train in Khandwa, Madhya Pradesh; however, when he woke up, he found himself in an empty compartment of a train thundering towards Kolkata where he spent a couple of weeks in a state of panic and hopelessness. Finally, he ended up in a local government adoption centre from where he was adopted by an Australian couple. Twenty five years later, Saroo felt the urge to trace his biological mother and see in what state she lived. Relentlessly, he used Google’s satellite feature to map the parts of the country that could have possibly been his own hometown. The search was a long and arduous one; nevertheless, the perseverance did pay. One eventful day, he met his mother; thereafter, he continued to keep in touch with her.

If technology can unite people with their loved ones, it can also make them distant. The unlimited variety of applications (apps) available to toddlers, teenagers and adults might have revolutionized their lives for the better, but these very apps have snatched away the joys of long naturewalks; they have encroached upon the time and space that people earlier used for physical interaction; they have drilled deep chasms of loneliness in the lives of countless numbers of people.

Simple pleasures of life include visiting friends and relatives, playing matches in open spaces, interacting with people in markets, public libraries and clubs. However, with the escalating rage of using apps like those for social media, playing virtual games, and homedelivery services, these joyous moments are fading into oblivion, and the pall of loneliness is getting heavier by the day.

Where are we heading to? Are we going to allow ourselves to be swamped by apps? Are we going to allow socialmedia to engulf us in a deluge of loneliness and isolation? Are we going to drive ourselves to situations that will ultimately demand mental and physical therapies to regain normalcy? Do we not know that physical interaction is as essential for mental health as food and water is for physical health?

Earlier, social isolation was mostly experienced by some of the elderly people who were devoid of an occupation, and bereft of company of their loved ones. Unfortunately today, an unhealthy solitude prevails among numerous children, teenagers and adults too; subsequently, there is an alarming increase in the demand for mental health therapy practitioners.

The necessity of engaging psychologists in schools and colleges is evidently on the rise. The psychologists are required to identify and address the learning and behavioral needs of students who approach them for guidance; moreover, if required, the professionals are expected to help them in strengthening their emotional, social and academic skills.

Regardless how alarming the situation might be, it is never too late. If people revert to the earlier trend of shopping off-line, going for naturewalks, playing outdoors games, and catching up with friends in their homes or cafés more frequently, they can keep their heads firmly well above the ocean of loneliness.

6. From the passage it is evident that Saroo’s desire to find his mother

(A) Ended up being a distant dream.

(B) Inspired him to use Google’s satellite feature intermittently.

(C) Waned as time went by.

(D) Did not slacken till he succeeded.

7. In the sentence ‘these very apps have snatched away the joys of long nature-walks;’ the author has

(A) Satirized nature

(B) Metaphorized apps

(C) Personified apps

(D) None of the above

8. From the passage one can conclude that

(A) It is impossible for people to reduce the usage of apps.

(B) There is a direct correlation between loneliness and excessive usage of social- media apps.

(C) The usage of technology is as essential for mental-health as food and water is for physical health.

(D) All senior citizens are lonely because they are not tech-savvy.

9. From the passage it can be inferred that presently in many educational institutions

(A) The number of teachers who pass the buck to psychologists is on the rise.

(B) Special emphasis is being laid on the mental and emotional health of the students.

(C) The usage of educational apps is being discouraged significantly.

(D) All the students feel the need to be counseled by psychologists.

10. In the concluding paragraph of the given passage, the writer’s tone can be best described as

(A) Optimistic

(B) Despairing

(C) Laudatory

(D) Apologetic

III. “Wash! Wash! Wash your hands! ” That’s been the safety-mantra ever since the pandemic COVID-19 began swamping the world. Undoubtedly, washing hands has proven to be the best way to keep germs at bay. Unfortunately, the medical practitioner who first promoted the importance of this simple activity was subjected to intense humiliation, and ultimately declared insane!

Ignaz Semmelweis was a Hungarian doctor. In 1847, as an obstetrician, he was disturbed that post-delivery, almost every third woman died of an unexpected malady. He observed that as a part of the set routine, medical students and doctors would examine and study the corpses in the mortuary, and then come for rounds to the maternity wards. Here, without washing their hands, they would examine expectant mothers. After making numerous hypothesis and observations, he was convinced that when doctors washed their hands before examining the women in the ward, the number of deaths due to serious infection declined. He shared his observations with his colleagues and many others working in the field of medicine, but unfortunately he could not provide any concrete evidence to his theory. Sadly, due to the vehement criticism that he received, he went into depression. Furthermore, Ignaz strived to prove his point so relentlessly that it led to the belief that he had lost his mind. In 1865, a doctor deceptively lured him into an asylum for the insane, and two weeks of the brutal treatment that was meted out to him by the attendants led to his untimely death. About twenty years later, when the world became more receptive to the works of scientists like Louis Pasteur and Joseph Lister, awareness regarding germs that cause diseases began to spread. This is the time when Ignaz was honoured with titles like Father of Hand Hygiene and Saviour of Mothers- an honour much too late!

Some of the most celebrated artists have earned fame much after their deaths. It is tragic that Vincent Van Gogh’s awe-inspiring work was labeled as strange and amateur by most of the critics of his time. It is believed that he sold only one or two painting in his lifetime, and that too for a meager amount. Today, every single painting of Vincent Van Gogh paintings is worth millions of dollars.

Franz Kafka was a proficient writer, but when he published a few pieces of his writings, he received immense criticism. Before his death in 1924, he handed over his unpublished novels and short stories to his friend Max Brod, and urged him to destroy them; however, Brod got the manuscripts published. Today, Franz is acclaimed as one of the major fiction writers of the twentieth century; the novels titled The Trial published in 1925, and The Castle published in 1926 are considered two of his masterpieces.

Perhaps, if humans were more tolerant and amenable to change, innovative concepts, theories and creations, the deserving would live to experience the glory and honour they rightfully deserve.

11. The main idea of the passage is that

(A) All original theories and works should receive unreserved acceptance.

(B) Many undeserving innovators have been honoured after their demise.

(C) Creativity must never be inhibited.

(D) Numerous innovators have found recognition and appreciation of their works posthumously.

12. From the passage it is evident that Dr. Ignaz’s theory was rejected because

(A) He could not substantiate it

(B) The doctors did not want him to regulate their work ethics

(C) He had been declared insane

(D) Joseph Lister and Louis Pasteur had already discovered germs

13. From the passage one can conclude that the art critics who Van Gogh’s works were

(A) Applauded, pessimistic.

(B) Censured, hypercritical.

(C) Denounced, tolerant.

(D) Acclaimed, rigid.

14. From the passage it can be inferred that Max Brod

(A) Was of the opinion that Franz had not reached out to the right critics

(B) Decried Franz’s writings

(C) Considered it unsacred to destroy any manuscript

(D) Appreciated and valued Franz’s works

15. The word relentlessly in the passage can be best replaced by the word

(A) Irresolutely

(B) Recklessly

(C) Unabatedly

(D) Unabashedly

IV. Cryptocurrencies are a terrible thing. They are the essence of a Ponzi scheme whose value is based entirely on a greater fool prepared to buy it. The promise of alchemy-turning lead into gold has bewitched humanity throughout the ages and cryptocurrencies are just the latest alchemy. Do not get me wrong, if rich people want to lose their money, in this or any other way, they should be allowed to do so. The rich should be the vanguards of new things in case something unforeseen and good falls out of them. But we need to protect those vulnerable consumers whose lives are such that almost any get-rich-quick schemes will be seductive, and seven out of 10 times, they will lose their life savings. Cryptocurrencies are today’s South Sea Bubble – one of the earliest recorded financial bubbles that took place in the 1720s’ Britain. Meme-based currencies like Dogecoin, Dogelon Mars and Doge Dash remind me of the infamous plan of one company during the South Sea Bubble to raise money “for carrying on an undertaking of great advantage; but nobody to know what it is.”

The cryptocurrency bubble is worse than tulip mania. Through the veil of technology, cryptocurrency enthusiasts are leaning on policy-makers to permit them to be exempt from regulation, privatize money, and make money so disconnected from the economy that it would reap financial disaster. There are many reasons to avoid financial disasters, but one of them is that they ratchet up poverty and inequality. The current money–credit system is not perfect, but like democracy, it is the worst system barring all the others. It has evolved from the ashes of the system cryptocurrency enthusiasts are trying to resurrect.

The current system is vulnerable to attack because money is little understood. Cryptocurrency enthusiasts have attracted a following based on the fiction that the central bank or government creates money and are busy debasing it in their self-interest. This is not the case, but then again, there is some overlap between cryptocurrency advocates, conspiracy theorists, and anti-vaxxers. The time has come for someone to stand up for the current fiat money system and explain that while it could be better still, it has been associated with far more growth, much more distributed, and has responded better to economic crisis than what came before.

In today’s money–credit system, banks create money when they issue a loan and place the loan’s proceeds into the account of their customers, creating a deposit. Money is, in fact, a tradable debt. The bank’s deposit can be used as cash because the bank is a regulated issuer of loans and deposit-taker, which gives the deposit credibility and convertibility. The central bank only influences the creation of money indirectly by its regulatory requirement that a proportion of the loans need to be funded by shareholder’s profits. They need to have skin in the game. Money creation then is based on thousands of separate decisions by loan officers and is more distributed than a centralized algorithm like Bitcoin. And its supply is determined by the private demand for loans, which means it is closely aligned to the economy.

16. Which of the following does best describe attitude of the author towards rich people?

(A) Concerned

(B) Assiduous

(C) Indifferent

(D) Sympathetic

17. Which of the following is true in the context of the passage?

(A) The author defends the current money–credit system.

(B) The author rejects the idea that the central bank or government creates money and are busy debasing it in their self-interest.

(C) The author backs the protection of poor from menace of cryptocurrencies.

(D) All the above

18. Which rhetorical device is employed in ‘cryptocurrencies are just the latest alchemy’?

(A) Antithesis

(B) Metaphor

(C) Personification

(D) Synecdoche

19. Which of the following does best describe the passage?

(A) Argumentative and explanatory

(B) Descriptive and argumentative

(C) Narrative and explanatory

(D) Expository and argumentative

20. What do the cryptocurrency enthusiasts rely on?

(A) Exemption from regulation

(B) Privatization of money

(C) Disconnection of money from the economy

(D) All the above

V. The fact that Gaia, in her monstrous avatar, decided to distribute fossil fuels very unevenly across the Earth has been central to the emergence of the world’s current geopolitical order. From a vitalist point of view, it could be said that the wars of the twentieth century were won as much by the fossilized energy of botanical matter as by particular groups of humans.

In the First World War Germany’s lack of oil put it at a huge disadvantage against the Allies, more or less ensuring its defeat. The shortage of oil effectively cancelled the technological advantages Germany enjoyed at the start of the war: despite having a large fleet, for instance, it was unable to use its navy effectively because its coal-burning ships needed to refuel every eleven days. Conversely, the assured supply of American oil conferred so great an advantage on Britain and France that “it could be fairly stated that the war was won for the Western allies by tankers.” Not for nothing was it said of the First World War that Britain, France, and the United States floated “to victory on a sea of oil.”

In the Second World War the shortage of oil was even more critical to the defeat of the Axis powers. The German Luftwaffe was forced to rely on synthetic fuels derived from coal, and these could not provide the high-octane energy that was necessary for high- compression aero engines: “it was largely due to the inferior engines in German aircraft that the Luftwaffe lost the Battle of Britain.” The shortage of oil also dictated Germany’s war strategy: it was in order to seize the oilfields of the Caucasus that the German army pushed eastward into the Soviet Union in 1942, leading to a defeat at Stalingrad from which it never recovered. Japan’s invasion of the Dutch East Indies was similarly forced by its lack of oil.

In short, over the course of the twentieth century access to oil became the central focus of global geopolitical strategy: for a Great Power, to be able to ensure or hinder the flow of oil was to have a thumb on the jugulars of its adversaries. In the first part of the twentieth century the guarantor of the flow of oil was Britain. After the Second World War, the baton was passed, along with a string of British naval bases, to the United States. The role of guarantor of global energy flows is still crucial to US strategic dominance and to its position as global hegemon.

Today, as Elizabeth DeLoughrey has pointed out, “US energy policy has become increasingly militarized and secured by the Navy, the largest oceanic force on the planet.” In the words of the historian Michael Klare, the Iraq War of 2003 marked the transformation of the US military into “a global oil protection service, guarding pipelines, refineries, and loading facilities in the Middle East and elsewhere.”

It is important to note that the strategic value of controlling oil flows is tangentially related to the US’s energy requirements. The period in which the American military was turning into “a global oil protection service” was one in which the US was well on its way to reducing its dependence on imported oil. The fact that the US is now self-sufficient in fossil fuels has in no way diminished the strategic importance of oil as an instrument for the projection of power- it is the ability to deny energy supplies to rivals that is strategically of central importance.

21. What is the central idea of the passage?

(A) Fossil fuels in war-making.

(B) Strategic value fossil fuels in US dominance in the world.

(C) Role of fossil fuels in modern geopolitical order.

(D) Distribution of fossil fuels in the world.

22. What was the cause of Germany’s defeat in the First World War?

(A) Germany’s shortage of oil

(B) Advantage of Britain and France

(C) Weaknesses of Germany’s navy

(D) All the above

23. Which of the following could be inferred from Michael Klare’s opinion on US military?

(A) The US military interferes with energy needs of other countries.

(B) The US energy policy has become increasingly militarized.

(C) The US has changed energy policy drastically.

(D) The US has fully understood the strategic value of controlling oil flows.

24. What does the phrase ‘tangentially related’ to mean?

(A) Related directly and in straightforward way

(B) Related closely and centrally

(C) Related only slightly and peripherally

(D) None of the above

25. What makes the US strategically dominating global hegemon?

(A) Capacity of the US to provide oil protection service, guarding pipelines, refineries, and loading facilities.

(B) Increasingly militarized energy policy of the US Navy, the largest oceanic force on the planet.

(C) The role of US as a guarantor of global energy flows.

(D) All the above

VI. The modern animal rights movement, which originated in the 1970s, may be understood as a reaction to dominant emphases within science and religion (principally, though not exclusively, Christianity). When the Jesuit Joseph Rickaby wrote in 1888 that “Brute beasts, not having understanding and therefore not being persons, cannot have any rights” and that we have “no duties of charity or duties of any kind to the lower animals as neither to stocks and stones”, he was only articulating, albeit in an extreme form, the moral insensitivity that has characterized the Western view of animals.

That insensitivity is the result of an amalgam of influences. The first, and for many years the most dominant, was the “other worldly” or “world denying” tendency in Christianity, which has, at its worst, denigrated the value of earthly things in comparison with things spiritual. Traditional Catholicism has divided the world into those beings that possess reason and therefore immortal souls, and those that do not. Christian spirituality has not consciously been at home with the world of non-human creatures-either animal or vegetable. Classic accounts of eternal life as found in Augustine of Hippo, Thomas Aquinas, or John Calvin make little or no reference to the world of animals. Animals, it seems, are merely transient or peripheral beings in an otherwise wholly human-centric economy of salvation.

The second idea-common to Christianity, Judaism, and Islam-is that animals, along with vegetables and minerals, exist instrumentally in relation to human beings; they are made for human beings, even belong to human beings, as resources in creation. This idea predates Christianity and is found notably in Aristotle, who argues that “since nature makes nothing to no purpose, it must be that nature has made them for the sake of man”. This idea, largely unsupported by scripture, was nevertheless taken over by Aquinas, who conceived of creation as a rational hierarchy in which the intellectually inferior existed for the sake of the intellectually superior.

Such instrumentalism, which features rationality as the key factor dividing human beings from “brute beasts,” has in turn buttressed the third influence, namely the notion of human superiority in creation. Human superiority need not, by itself, have led to the neglect of animal life, but when combined with the biblical ideas of being made “in the image of God” and God’s preferential choice to become incarnate in human form, some sense of moral as well as theological ascendancy was indicated. As a result, Christianity, and to a lesser extent Judaism, have been characterized historically by an overwhelming concern for humanity in creation rather than an egalitarian concern for all forms of God-given life. That humans are more important than animals, and that they self-evidently merit moral solicitude in a way that animals cannot, has become religious doctrine. Thus the Catechism of the Catholic Church maintains that “it is . . . unworthy to spend money on them [animals] that should as a priority go to the relief of human misery”.

These influences have in turn enabled and justified the scientific exploration of the natural world and specifically the subjection of animals to experimentation. Francis Bacon pursued his scientific investigations in the belief that humanity should “recover that right over nature which belongs to it by divine bequest”. René Descartes famously likened the movements of a swallow to the workings of a clock, and maintained that “There is no prejudice to which we are more accustomed from our earliest years than the belief that dumb animals think”.

26. Jesuit Joseph Rickaby’s articulation on animals may be termed as:

(A) Eco-centric view of animals (B) Anthropocentric view of animals

(C) Ethnocentric view of animals (D) Androcentric view of animals

27. According to the author, how did Christianity contribute to insensitivity of the West towards animals?

(A) It denigrated the value of earthly things in comparison with spiritual things.

(B) It divided the world into beings with and without reason.

(C) It propagated as if animals are transient or peripheral in human centric economy of salvation.

(D) All the above.

28. Which of the following is closest to the meaning of the word ‘instrumentalism’ as used in the passage?

(A) Pragmatism

(B) Idealism

(C) Egalitarianism

(D) None of the above

29. Which of the following is not true in the context of the passage?

(A) Western philosophy and science are both under the influence of religion.

(B) Western philosophical views on animals have been influenced by religious notions about them.

(C) Western religious notions on animals have justified subjection of animals to scientific experimentation.

(D) Some of the scientific views on animals have been influenced by religious notions about them.

30. It may be inferred from René Descartes’ view that

(A) It as irrational to assume that animals have awareness and some mental capacities.

(B) Animals are automata or they act mechanically.

(C) Neither (A) nor (B)

(D) Both (A) and (B)

Current Affairs and General Knowledge

VII. When we hear the name SPACE, only one organization comes to mind: the Indian Space Research Organization (ISRO). The Indian Space Research Organization (ISRO), located in Bengaluru, is the country’s first space agency. ISRO was founded in 1969 with the goal of developing and utilizing space technology for national development while also conducting planetary exploration and space science research. The space research operations began in India in the early 1960s, at a time when satellite applications were still in the experimental stages in the United States. Dr. Vikram Sarabhai, the founding father of India’s space programme, rapidly recognized the benefits of space technologies after the live transmission of the Tokyo Olympic Games across the Pacific by the American satellite ‘Syncom-3’ demonstrated the power of communication satellites. The ISRO has launched various spacecrafts like the Chandrayaan, Astrosat, Microsat, GSAT etc. The Government of India has also approved a regional spaceborne navigation system, which will consist of seven satellites. Out of these, four of them will be placed in geosynchronous inclined orbit of 29° relative to the equatorial plane. Such an arrangement would mean all seven satellites would have continuous radio visibility with Indian control stations.

31. Name the first dedicated mission launched by ISRO for studying the celestial sources in X-ray, optical and UV spectral bands simultaneously.

(A) Amazonia

(B) Astrosat

(C) Gaganyaan – 1

(D) Lunar Polar Exploration Mission

32. APPLE, the first communication satellite of ISRO was launched from:

(A) Satish Dhawan Space Centre, Sriharikota

(B) TERLS, Thiruvananthapuram

(C) Kourou, French Guiana

(D) SSLV Launching Station, Tamil Nadu

33. Name the spacecraft that has been successfully sent into the space to probe into the planets in the solar system.

(A) Sputnik 19

(B) Cosmos 482

(C) Pioneer-E

(D) Mariner 10

34. The first successful Nuclear Bomb test conducted by India in 1974, is called the:

(A) Prithvi

(B) Pokhran-II

(C) Smiling Buddha

(D) Surya

35. NaviC covers India and region extending:

(A) 1,500 Km

(B) 2,000 Km

(C) 7,000 Km

(D) 1,000 Km

VIII. On December 26, 2021, for the first time since the present government came to power in 2014, the Union Home Ministry constituted a panel led by its officers to review the withdrawal of the Armed areas. The Act was amended in 1972 and the powers to declare an area as “disturbed” were conferred concurrently upon the Central Government along with the States. Currently, the MHA issues periodic “disturbed area” notification to extend AFSPA only for Nagaland and Arunachal Pradesh, where it is applicable in the districts of Tirap, Changlang, Longding and areas falling under Namsai and Mahadevpur police stations bordering Assam. The notification for Manipur and Assam is issued by the State Governments. Tripura revoked the Act in 2015 and Meghalaya was under AFSPA for 27 years, until it was revoked by the MHA from April 1, 2018. The Act was implemented in a 20-km area along the border with Assam. Jammu and Kashmir has a separate J&K Armed Forces (Special Powers) Act, 1990.

36. Power to notify parts of or the whole of a State or a Union Territory as a ‘disturbed area’, under the Armed Forces (Special Powers) Act, 1958 is vested with:

(A) Governor of any State

(B) Administrator of a Union Territory

(C) Central Government

(D) All of the above

37. AFSPA was introduced in Meghalaya in the year:

(A) 1995

(B) 1999

(C) 1991

(D) 1989

38. Consider the following statements about the Armed Forces (Special Powers) Act and mark the correct option.

(A) Detractors and Human Rights Organizations, as well as many sections of civil society, argue that the Act often leads to excesses and require checks and balances or could alienate the people instead of integrating them with the main stream.

(B) Irom Chanu Sharmila, an activist from Manipur, became ansensitizing figure symbolizing the protest against AFSPA after she sat on an indefinite fast in 2000, demanding its repeal and ended it only in August 2016.

(C) Both (A) and (B) are correct

(D) None of the above

39. In 2005, a government-appointed five-member committee, recommended that AFSPA be repealed. It suggested that the Unlawful Activities (Prevention) Act could be suitably amended to deal with terrorism. It made this recommendation as it felt that the AFSPA created an impression that the people of the Northeast States were being targeted for hostile treatment. Who headed this committee?

(A) B. P. Jeevan Reddy

(B) Abhishek Singhvi

(C) Soli Sorabjee

(D) Ram Jethmalani

40. AFSPA is currently in force in:

(A) Arunachal Pradesh and Assam

(B) Nagaland and Manipur

(C) Jammu and Kashmir

(D) All of the above

IX. One of the justifications of Russia-Ukraine war Russian leader claimed that military action was necessary to stop Ukrainian attacks on the two break away regions of Donetsk and Lugansk, which Russia recognized as sovereign states. President of Russia claimed that Russia could come under attack by Ukrainian far right government, unless their influence in the country is diminished. He accused Western Nations of arming Kyiv against Russia. After being ordered by Russia’s leader to invade the capital of Ukraine, Russian troops moved in several directions. As the first targets were hit, airports and the military HQs were located near cities across Ukraine, then tanks and troops rolled into the country from the north, east, and south. The war has demolished most of the cities of the Ukraine and caused irreparable damage to the humanity.

41. Ukraine was part of which country during 1922-1991?

(A) Germany

(B) Poland

(C) USA

(D) USSR

42. Who is the President of Ukraine?

(A) Volodymyr Zelenskyy

(B) Vladimir Zelenskyy

(C) Volodymyr S. Zelenskyy

(D) Vladimir Zelensky

43. Which is the currency of Ukraine?

(A) Guilder

(B) Zloty

(C) Ruble

(D) Hryvnia

44. The President of Ukraine was formerly:

(A) Weather Forecaster

(B) Navy Officer

(C) Comedian

(D) TV News Reader

45. Which operation was launched by the Indian Government to bring back Indians from Ukraine during Russia-Ukraine war?

(A) Operation Kyiv

(B) Operation Ganga

(C) Operation Rakshak

(D) Operation East Star

X. Every second patent granted in India between 2016-2021 is related to green technology and a quarter of the green technology patents are concerning alternative energy production, coinciding with the Centre’s efforts on “enhanced use of green technology for boosting economy and encouraging consumers to use products produced through use of such technology”. Data from the Ministry of Commerce and Industry shows that more than 91,500 patents were granted between 2016-17 and 2020-21, while the data for 2021-22 was not immediately available. However, going by the trend in the past two years, India is expected to clear at least 25,000 more applications this year. Separate data from the ministry shows that between 2016-17 and 2021-22 (up to January), 61,186 patents related to green technologies have been granted in the said period. Of these, 90% are for technologies concerning waste management and alternative energy production- 38,837 or 63% of them are related to waste management and more than 16,000 or 26% are for alternative energy production. The rest of the green technology patentsare for energy conservation (2,555),transportation technologies (2,481),nuclear power generation (1,079),agriculture-and-forestry (161),and others (69).

46. With which country India entered into a ‘Green Strategic Partnership’ in September 2020?

(A) Poland

(B) Greece

(C) Denmark

(D) South Korea

47. In pursuance of the United Nations Sustainable Development Goals (SDG), India aims to be energy independent by the year

(A) 2047

(B) 2040

(C) 2045

(D) 2057

48. According to the Global Innovation Index 2021 published by the World Intellectual Property Organization (WIPO), India is ranked out of 132 countries.

(A) 45th

(B) 46th

(C) 47th

(D) 48th

49. In January 2022, the Union Cabinet approved the second phase of the Green Energy Corridor (GEC) in India with the objective to facilitate

(A) Grid integration and power evacuation of Renewable Energy (RE) Power Projects in seven States.

(B) Ecologically sustainable growth by increasing carbon footprint.

(C) Ecologically sustainable production of fossil fuels.

(D) Spreading awareness regarding Renewable Energy (RE) Power Projects.

50. The IRENA is an intergovernmental organization that supports countries in their transition to a sustainable energy future and serves as the principal platform for international co-operation on renewable energy. IRENA stands for:

(A) Intergovernmental Renewable Energy Association

(B) International Renewable Energy Association

(C) Intergovernmental Renewable Energy Agency

(D) International Renewable Energy Agency

XI. The Government may defer the deadline for companies to deposit the unspent portion of their Corporate Social Responsibility (CSR) funds into specified bank accounts in a move aimed at providing some relief in the fight against the COVID-19 and subsequent restrictions. Companies are required to spend at least 2% of their average net profit of the preceding within three financial years on CSR. The Ministry has allowed firms to undertake projects on an ongoing basis on the condition that any unspent amount must be deposited with a scheduled bank within 30 days of the end of the financial year. “Considering the current crisis, we request your esteemed office to consider relaxation by providing extension of the timelines up to June 30, 2021,” the Institute of Company Secretaries of India said in a letter to the Ministry. Experts said that while lockdowns and restrictions are less stringent than last year and companies have adapted to working online, many professionals or their families have been affected by the infection, leaving teams short-staffed.

51. Which of the following is the Corporate Social Responsibility (CSR) initiative of the erstwhile Facebook India to promote women entrepreneurship?

(A) Shakti

(B) Pragati

(C) Lean In

(D) Marching Forward

52. In 2019, which of the following Committees recommended that CSR expenditure should be made tax deductible expenditure?

(A) Company Laws Committee

(B) High Level Committee on Reform of Indian Company Law

(C) High Level Committee on Corporate Social Responsibility

(D) Committee for Reforms in Corporate Social Responsibility in India

53. CAWACH is an initiative by the National Science and Technology Entrepreneurship Development Board, Department of Science and Technology, Government of India stands for the:

(A) Centre for Alleviating Waning Healthcare Companies

(B) Centre for Augmenting Wound-up Healthcare Companies

(C) Centre for Augmenting WAR with COVID-19 Health Crisis

(D) Centre for Alleviating Withering Healthcare Companies

54. On January 22, 2021 the Companies (Corporate Social Responsibility Policy) Amendment Rules, 2021 were notified by the:

(A) Ministry of Finance

(B) Ministry of Social Justice and Empowerment

(C) Ministry of Law and Justice

(D) Ministry of Corporate Affairs

55. The Ministry of Corporate Affairs has instituted an award to select companies to recognize corporate initiatives in the area of Corporate Social Responsibility (CSR) to achieve inclusive growth and inclusive and sustainable development. The name of the award is:

(A) National Corporate Social Responsibility Awards

(B) National Awards for Excellence in Social Responsibility

(C) National Awards for Corporate Excellence in Social Responsibility

(D) National Awards for Excellence in Inclusive Growth and Sustainable Development

XII. With the announcement of Drone Shakti in the Union Budget, the industry got a massive push after the liberalization of the Drone Rules in 2021. The Government stated that start- ups will be promoted to facilitate Drone Shakti, with ‘drone as a service’.

“The current Government has taken a serious paradigm shift on drone technology and Drone Shakti announcement of the current FY budget proves the government’s clear vision and focus towards this emerging industry. Drone Shakti and Kisan Drones will definitely help get this technology to impact common people on the grassroots level at a massive scale,” according to the founder and CEO of a prominent aerospace quoted in a business magazine. The country is set to witness the use of large, unmanned aircraft systems weighing more than 150 kilograms across the sectors. Kisan Drones are already being used for crop assessments, land records, spraying of insecticides, and are expected to boost a wave of technology in the Agri and farming sector. Drones are also being used in surveillance systems for Railway Security. In India, drones were also deployed to deliver COVID-19 vaccines.

56. COVID-19 vaccines were delivered by drones as ICMR-led pilot project to:

(A) Ladakh

(B) Katra

(C) Dantewada

(D) Manipur

57. Which of the following is the online platform hosted by the Directorate General of Civil Aviation for various activities related to the management of drone activities in India?

(A) e-Drone Shakti

(B) Digital Sky platform

(C) Aakash Shakti

(D) Digital Drone platform

58. Which of the following refers to the phenomenon of restricting the movement of drones within a defined airspace?

(A) Geo-fencing

(B) Drone-fencing

(C) Sky-fencing

(D) Air-fencing

59. The Drone Rules, 2021 have been made in supersession of which of the following rules?

(A) The Unmanned Aircraft System Rules, 2020

(B) The Aircraft Rules, 2020

(C) The Unmanned Aircraft System Rules, 2021

(D) The Manned Aircraft Rules, 1934

60. Which autonomous body is responsible for national accreditation structure for drones?

(A) Drone Council of India

(B) Unmanned Aircraft Council of India

(C) Manned Aircraft Council of India

(D) Quality Council of India

XIII. The 21st century has seen an overall shift in India’s policy outlook and also the global outlook towards India. Coherent policy initiatives and effective implementation on the ground have ensured a positive growth profile. The biggest shift has been the recognition of the maritime sector. Somehow, post-independence, we as a nation became sea blind and completely undermined our maritime potential. Initially, the Indian Navy came out with their Military

Maritime Strategy in 2007, titled “Freedom to use the Seas”. They kept on upgrading their vision document on regular intervals with latest being declared in 2015. The Security And Growth for All in the Region (SAGAR) vision declared by the Government of India in May 2015 has been a massive game changer on multiple fronts. It integrates the geopolitical and geostrategic realities to domestic requirements. The security concerns and the blue economic opportunities are comprehensively addressed along with the diplomatic leverage and reviving the rich maritime heritage.

The SAGAR vision has been backed by aggressive push by the Government of India to drive mega projects with massive human resource requirements.

61. Which of the following is the flagship programme of the Ministry of Shipping, Government of India, to promote port-led development in the country through harnessing India’s 7,500 km long coastline, 14,500 km of potentially navigable waterways and strategic location on key international maritime trade routes?

(A) Sagarmala Programme

(B) Sagarshakti Programme

(C) Jalshakti Programme

(D) Jalmala Programme

62. Which of the following is the initiative of the Ministry of Earth Sciences (MoES), Government of India, to explore marine resources and develop deep sea technologies for sustainable use of ocean resources?

(A) Sagarmanthan Mission

(B) Samudramanthan Mission

(C) Deep Ocean Mission

(D) Deep Ocean and Sea Technology Mission

63. ISA is a United Nations body regulating the exploration and exploitation of marine non-living resources of oceans in international waters. ISA stands for:

(A) International Sea Authority

(B) International Sea Association

(C) International Seafaring Association

(D) International Seabed Authority

64. Under Mission SAGAR, which Indian Naval Ship was deployed by the Indian Navy to provide humanitarian and medical assistance to Maldives, Mauritius, Seychelles, Madagascar and Comoros?

(A) INS Kesari

(B) INS Vikrant

(C) INS Viraat

(D) INS Vikramaditya

65. Which of the following is India’s first manned ocean mission which was launched at Chennai?

(A) Samudrashakti

(B) Samudrayan

(C) Sagaryan

(D) Sagarshakti

Legal Reasoning

XIV. Writ is a public law remedy. It refers to a formal, written order issued by a judicial authority directing an individual or authority to do or refrain from doing an act. The High Court, while exercising its power of judicial review, does not act as an appellate body. It is concerned with illegality, irrationality and procedural impropriety of an order passed by the State or a Statutory Authority. A High Court is empowered to issue directions, orders or writs for the enforcement of Fundamental Rights and for any other purpose. The writ jurisdiction of High Court is discretionary and equitable. Writ of mandamus is issued by a court commanding a public authority to perform a public duty belonging to its office. It can be issued only when a legal duty is imposed on the authority and the petitioner has right to compel the performance of such duty. Writ of mandamus is requested to be issued, inter alia, to compel performance of public duties which may be administrative, ministerial or statutory in nature. A writ of mandamus may be issued in favour of a person who establishes a legal right in himself. It may be issued against a person who has a mandatory legal duty to perform, but has failed or has neglected to do so. Such a legal duty emanates by operation of law. The writ of mandamus is most extensive in regards to its remedial nature. The object of mandamus is to prevent disorder emanating from failure of justice and is required to be granted in all cases where law has established no specific remedy.

66. The government of a state made a rule to make it discretionary to grant dearness allowance to its employees. One of the employees filed a writ petition seeking a mandamus to compel the government to grant dearness allowance. In the given situation, which of the following statements is true?

(A) Writ of mandamus cannot be granted as no legal duty was imposed on the government to grant dearness allowance.

(B) Writ of mandamus cannot be granted as a statutory right was conferred on the employee to receive dearness allowance.

(C) Writ of mandamus can be granted as it is a public law remedy.

(D) Writ of mandamus can be granted as it is a discretionary remedy.

67. A public-spirited citizen filed a writ petition seeking a mandamus to compel the government to make a law to curb the ill-effects of climate change. In the given situation, which of the following statements is true?

(A) Writ of mandamus can be granted as it is an equitable remedy.

(B) Writ of mandamus cannot be granted as no legal duty is imposed on the government to pass a law to curb the ill-effects of climate change.

(C) Writ of mandamus can be granted as it is a discretionary remedy.

(D) Writ of mandamus cannot be granted as there is no violation of fundamental right.

68. Mr. A and Mr. B are parties to a contract of sale of goods. Upon breach of contractual obligations by Mr. A, Mr. B filed a writ petition seeking a mandamus to compel Mr. A to perform his obligations under the contract. In the given situation, which of the following statements is true?

(A) Writ of mandamus can be granted as it is a discretionary remedy.

(B) Writ of mandamus cannot be granted as there is no violation of fundamental right.

(C) Writ of mandamus cannot be granted as Mr. B’s right under the contract is a private right.

(D) Writ of mandamus can be granted as there is no statutory duty imposed on Mr. A to fulfill his contractual obligations.

69. A licensing officer is under a statutory duty to issue a license to an applicant who fulfils the conditions prescribed for the issue of such license. Mr. X, an applicant, fulfilled all the conditions prescribed for the issue of such license, but his application for issuance of license was rejected by the licensing officer. In the given situation, which of the following statements is correct?

(A) Writ of mandamus can be granted compelling the license officer to issue the license.

(B) Writ of mandamus cannot be granted compelling the license officer to issue the license as there is no violation of fundamental right.

(C) Writ of mandamus can be granted as it is the discretion of the licensing officer to grant license.

(D) Writ of mandamus cannot be granted compelling the license officer to issue the license as there is no violation of public duty.

70. Which of the following statements is incorrect?

(A) Writ of mandamus may be issued in favour of a person who establishes the existence of a legal right.

(B) Writ of mandamus may be issued against a person or authority who has a mandatory duty to perform but has failed or has neglected to do so.

(C) Writ of mandamus is purported to prevent disorder emanating from failure of justice.

(D) Writ of mandamus may be requested to be issued to compel performance of private duties which may be administrative, ministerial or statutory in nature.

XV. To maintain the secular character of the Indian polity, not only does the Constitution of India guarantee freedom of religion to individuals and groups, but it is also against the general policy of the Constitution of India that any money be paid out of the public funds for promoting or maintaining any particular religion. Accordingly, it is provided in the Constitution of India that no person shall be compelled to pay any taxes, the proceeds of which are specifically appropriated in payment of expenses for the promotion or maintenance of any particular religion or religious denomination. This does not prohibit the State from enacting a law to incur expenses for the promotion or maintenance of any particular religion or religious denomination, but by such law, no person can be compelled to pay any tax, the proceeds of which are to be so utilized. This, however, does not invalidate levy of a fee to provide some service. Thus, a fee can be levied on pilgrims to a religious fair to meet the expenses of the measures taken to safeguard the health, safety and welfare of the pilgrims. Such fee levied by a State will be valid because the object of such contribution is not to foster or preserve religion, but to control secular administration of religious institutions.

71. Mr. A, a tax-payer of various taxes levied by the State Government, filed a writ petition for issuance of a writ of mandamus directing the State to forbear from spending any amount from the public funds of the state for renovation of water tanks belonging to a Hindu temple in the State. These tanks were used by the general public, irrespective of their religious affiliation, for bathing and drinking purposes. In the given situation, which of the following statements is correct?

(A) The State is promoting or maintaining the Hindu religion.

(B) The State cannot be said to be promoting or maintaining the Hindu religion.

(C) The State is compelling citizens to pay tax for promotion of Hindu religion.

(D) The State is compelling citizens to pay tax for maintenance of Hindu religion.

72. Communal riots between religion ‘A’ and religion ‘B’ resulted in the destruction of places of worship of both religions in a State. As a result, the State Government utilized public funds for restoring the places of worship of both religions ‘A’ and ‘B’. In the given situation, which of the following statements is correct?

(A) The State Government’s act is violative of the Constitution of India as it is promoting or maintaining particular religions.

(B) The State Government’s act is not violative of the Constitution of India as it is not promoting or maintaining any particular religion.

(C) The State Government’s act is violative of freedom of religion as it condones communal violence.

(D) The State Government’s act is not violative of the Constitution of India as it is promoting or maintaining a particular religion.

73. Communal riots between religion ‘A’ and religion ‘B’ resulted in the destruction of a place of worship of religion ‘B’ in the State. As a result, the State Government utilized public funds for restoring the places of worship of religion ‘B’. In the given situation, which of the following statements is correct?

(A) The State Government’s act is violative of the Constitution of India as it is promoting or maintaining a particular religion.

(B) The State Government’s act is not violative of the Constitution of India as it is promoting or maintaining a particular religion.

(C) The State Government’s act is not violative of the Constitution of India as it is not promoting or maintaining a particular religion.

(D) The State Government’s act is violative of freedom of religion as it condones communal violence.

74. A State Government passed a law making it mandatory for all residents of the State to pay a hefty ‘pilgrimage tax’ so as to aid the State in organizing a popular religious pilgrimage in that State. The said pilgrimage is undertaken only by followers of religion ‘X’ and draws a large number of followers of religion ‘X’ from all over the country to the state every year. In the given situation, which of the following statements is correct?

(A) The State Government’s act is violative of the Constitution of India as it is not promoting or maintaining religion ‘X’.

(B) The State Government’s act is not violative of the Constitution of India as it is not promoting or maintaining religion ‘X’.

(C) The State Government’s act is violative of the Constitution of India as it is compelling citizens to pay tax for promoting or maintaining religion ‘X’.

(D) The State Government’s act is not violative of the Constitution of India as it is a measure to safeguard the health, safety and welfare of the pilgrims.

75. The object of a State Legislation is “to provide for the better administration and governance of certain Hindu religious endowments”, where ‘religious endowment’ means the property belonging to or given or endowed for the support of temples. The State Legislation vests the supervision of public temples in a statutory authority, i.e., ‘Commissioner of Hindu Religious Endowments’. For the purpose of meeting the expenses of the Commissioner and his staff, every Hindu temple in the State is required under provisions of the State Legislation, to pay an annual contribution at certain percentage of their annual income. In the given situation, which of the following statements is correct regarding the annual contribution provided under the State Legislation?

(A) It is not violative of the Constitution of India because its object is the proper administration of religious trusts and institutions.

(B) It is not violative of the Constitution of India because its object is fostering of Hindu religion.

(C) It is violative of the Constitution of India because its object is fostering of Hindu religion.

(D) It is violative of the Constitution of India because its object is not inclusive of administration of religious endowments belonging to all religions.

XVI. The Constitution of India guarantees to all its citizens certain fundamental freedoms, which are recognized as their fundamental rights. However, these fundamental freedoms guaranteed by the Constitution of India are not absolute as no right can be. Each of these fundamental rights is liable to be controlled, curtailed and regulated to some extent by laws made by the Parliament or the State Legislatures. Accordingly, the Constitution of India lays down the grounds and the purposes for which a legislature can impose ‘reasonable restrictions’ on the rights guaranteed to citizens. The State cannot travel beyond the contours of these reasonable restrictions in curbing the fundamental rights guaranteed to citizens. While determining the constitutional validity of a restriction imposed on a fundamental right by a legislation, the Court is not concerned with the necessity of the restriction or the wisdom of the policy underlying it, but only whether the restriction is in excess of the requirement, and whether the legislature has overstepped the Constitutional limitations. Two of the fundamental rights guaranteed to every citizen of India are- the right to move freely throughout the territory of India and the right to reside and settle in any part of India. However, the State may impose reasonable restrictions on these rights by law, in the interests of the general public or for the protection of the interests of any Scheduled Tribes.

76. A law was enacted by the Parliament of India which consisted of a provision making it mandatory for every person riding a two-wheeler in India, to wear a helmet, failing which such person was made liable to a fine. Mr. X, a citizen of India, was fined for violation of the said provision. Mr. X challenged the constitutional validity of the said provision. In the given situation, which of the following statements is correct?

(A) The provision is violative of the Constitution of India because it is a restriction on the freedom to move freely throughout the territory of India.

(B) The provision is not violative of the Constitution of India because it is a reasonable restriction on the freedom to move freely throughout the territory of India.

(C) Mr. X’s fundamental right to move freely throughout the territory of India is violated.

(D) Both (A) and (C).

77. A group of Indian students of XYZ University located in New Delhi, India posted on social networking sites that they would hold a demonstration outside the university campus, protesting against a recently passed law which made it compulsory for university students to wear uniforms while attending classes. The students further threatened to “use whatever means necessary” to “stop the oppression of students”. Therefore, the State Authorities placed barricades around the university campus in order to restrict movement of the students carrying out the demonstration and ensuring that the demonstration does not turn violent. In the given situation, which of the following statements is correct regarding the act of placing of barricades by State Authorities?

(A) The act is violative of the Constitution of India because it is a restriction on the freedom to move freely throughout the territory of India.

(B) The act is not violative of the Constitution of India because it is a reasonable restriction in the interests of general public.

(C) The act is violative of the Constitution of India because it is restriction in the interest of students.

(D) The act is not violative of the Constitution of India because it is a reasonable restriction in the interest of morality.

78. The appropriate authority in a State passed an externment order against Mr. A, a citizen of India. The externment order prohibited Mr. A, from residing within the State, from the date specified in such order. The externment order was passed by virtue of powers conferred on the appropriate authority by law, and the constitutional validity of this law had been upheld by the Supreme Court of India. The externment order was passed on the ground that Mr. A was found to be frequently engaged in illegal business of narcotic drugs and was also involved in several cases of riot and criminal intimidation. In the given situation, which of the following statements is correct regarding the externment order?

(A) It is a reasonable restriction on Mr. A’s fundamental right of free movement throughout the territory of India.

(B) It is an unreasonable restriction on Mr. A’s fundamental right of residence and settlement in any part of India.

(C) It is violative of Mr. A’s fundamental right of free movement throughout the territory of India.

(D) It is an unreasonable restriction on Mr. A’s fundamental right of free movement throughout the territory of India.

79. Mr. Z, a citizen of India, was issued a passport on June 1, 2020 by the Passport Office. Mr. Z was due to travel to Spain on July 15, 2021. On July 11, 2021, Mr. Z received a letter from the Regional Passport Officer intimating him that it was decided by the Government of India to seize his passport “in public interest”. Mr. Z was required to surrender his passport within seven days of the receipt of that letter. In the given situation, which of the following statements is correct?

(A) Mr. Z can challenge the letter on the ground that it is violative of his fundamental right of free movement throughout the territory of India.

(B) Mr. Z can challenge the letter on the ground that it is violative of his fundamental right to reside and settle in any part of India.

(C) Mr. Z can challenge the letter on the ground that it is violative of the law relating to passports in India.

(D) Mr. Z cannot challenge the letter on the ground that it is violative of his fundamental right(s) of free movement throughout the territory of India and/or to residence and settlement in any part of India.

80. Which of the following statements is incorrect?

(A) Fundamental right to movement and residence in any part of India are sacrosanct and are guaranteed to all citizens.

(B) Fundamental right to movement and residence in any part of India are sacrosanct, but are guaranteed subject to reasonable restrictions on such rights.

(C) Reasonable restrictions may be imposed, on fundamental rights to movement and residence in any part of India, by law.

(D) The constitutional validity of a law imposing reasonable restrictions on fundamental rights can be challenged by a citizen before the legislature.

XVII. Where a spouse contracts a second marriage while the first marriage is still subsisting, the spouse would be guilty of the offence of bigamy under the penal law in India, if it is proved that the first as well as the second marriages were legally valid, i.e., all the necessary ceremonies required by law or by custom have been performed at the time of contracting the marriages. According to the penal law in India, if a person, who has a living husband or wife, marries again, then such person is liable to be punished with imprisonment up to seven years along with a fine for committing the offence of bigamy. Although the penal law of India is applicable to all citizens irrespective of their religious affiliations, an exception to the offence of bigamy may be created by the law relating to marriage applicable to followers of a particular religion. Under the Hindu law relating to marriage, bigamy is not permitted. If a Hindu wife files a criminal complaint against her husband on the ground that during the subsistence of her marriage, her husband had married a second wife by converting into another religion which legally permits having more than one wife, then her husband is liable to be punished for the offence of bigamy. Further, the Hindu law relating to marriage also provides that the punishment for offence of bigamy as provided in the penal law of India would be applicable to marriage between two Hindus.

81. Mr. A, a Hindu male, has been married to Ms. B, a Hindu female. Their marriage was solemnized as per Hindu rites and ceremonies. After his marriage to Ms. B, Mr. A underwent religious conversion into a religion ‘X’ which legally permits males to have two wives. Thereafter, Mr. A got married to Ms. C, a female belonging to religion ‘X’, in compliance with all the legal requirements of contracting a valid marriage under religion ‘X’. In the given situation, which of the following statements is true?

(A) As Mr. A married Ms. C, the marriage of Mr. A and Ms. B has become invalid.

(B) As Mr. A is not a Hindu, the marriage of Mr. A and Ms. B has become invalid.

(C) Mr. A’s marriage with Ms. C has not affected the validity of his marriage with Ms. B.

(D) Both (A) and (B).

82. Mr. A, a Hindu male, has been married to Ms. B, a Hindu female. Their marriage was solemnized as per Hindu rites and ceremonies. After his marriage to Ms. B, Mr. A underwent religious conversion into a religion ‘X’ which legally permits males to have two wives. Thereafter, Mr. A got married to Ms. C, a female belonging to religion ‘X’, in compliance with all the legal requirements of contracting a valid marriage under religion ‘X’. Ms. B filed a criminal complaint against Mr. A for committing the offence of bigamy. In the given situation, which of the following statements is true?

(A) Mr. A is liable to be punished according to the Hindu law relating to marriage.

(B) Mr. A is liable to be punished according to the penal law of India.

(C) Mr. A has not committed the offence of bigamy.

(D) Both (A) and (B).

83. Mr. A, a Hindu male, has been married to Ms. B, a Hindu female. Their marriage was not solemnized as per Hindu rites and ceremonies or any other custom, but was performed by seeking blessings of their family members. After his marriage to Ms. B, Mr. A underwent religious conversion into a religion ‘X’ which legally permits males to have two wives. Thereafter, Mr. A got married to Ms. C, a female belonging to religion ‘X’, in compliance with all the legal requirements of contracting a valid marriage under religion ‘X’. Ms. B filed a criminal complaint against Mr. A for committing the offence of bigamy. In the given situation, which of the following statements is true?

(A) Mr. A has committed the offence of bigamy because he married again during the subsistence of the first marriage.

(B) Mr. A has not committed the offence of bigamy because his first marriage is not valid.

(C) Mr. A has committed the offence of bigamy because he underwent religious conversion in order to contract a bigamous marriage.

(D) Mr. A has not committed the offence of bigamy because his second marriage is not valid.

84. Mr. A, a male belonging to religion ‘P’, has been married to Ms. B, a female belonging to religion ‘P’. Their marriage was solemnized in compliance with all the legal requirements of contracting a valid marriage under religion ‘P’. Monogamy is espoused as a cherished value by the followers of religion ‘P’ and provided as a pre-condition for a valid marriage for the followers of the religion. After his marriage to Ms. B, Mr. A underwent religious conversion into a religion ‘Q’ which legally permits males to have two wives. Thereafter, Mr. A got married to Ms. C, a female belonging to religion ‘Q’, in compliance with all the legal requirements of contracting a valid marriage under religion ‘Q’. Ms. B wife filed a criminal complaint against Mr. A for committing the offence of bigamy. In the given situation, which of the following statements is true?

(A) Mr. A has committed bigamy according to the Hindu law relating to marriage.

(B) Mr. A has committed bigamy according to the penal law of India.

(C) Mr. A has committed bigamy according to the law relating to marriage of religion ‘P’.

(D) Both (B) and (C).

85. Which of the following statements is incorrect?

(A) Marrying again during lifetime of husband or wife is a pre-condition for performing a valid Hindu marriage.

(B) Religious conversion is not a defence for the offence of bigamy under the penal law of India.

(C) Bigamy is an offence under the penal law of India.

(D) Offence of bigamy can be committed according to the provisions of Hindu law relating to marriage.

XVIII. A special marriage, i.e., a marriage between persons from two different religious affiliations can be legally contracted in India under the provisions of the law relating to special marriages. The law relating to special marriages provides for the registration of such marriages and for divorce in such cases. One of the modes in which a special marriage can be legally terminated is through divorce by mutual consent of parties to the marriage. In order to obtain a divorce by mutual consent, both the parties to the special marriage are required to jointly present a petition for divorce to the district court on the ground that they have been living separately for one year or more, that they have not been able to live together and that they have mutually agreed that the marriage should be dissolved. At least six months after the presentation of such petition, but not later than eighteen months after the presentation of such petition, the district court, after hearing the parties and after making the necessary inquiry, and being satisfied that the marriage is a ‘special marriage’, and that the claims made in the petition are true, shall declare the marriage to be dissolved. Further, the personal presence of the parties before the district court at the time of presenting the joint petition for divorce is not mandatory, as the parties can satisfy the court even by affidavit that the requirements for granting divorce on mutual consent are fulfilled.

86. Ms. A, a Hindu female and Mr. B, a Christian male, got married as per the law relating to special marriages in January 2018. On January 26, 2021, Ms. A and Mr. B jointly presented a petition for divorce by mutual consent before the district court on the ground that they have been inflicting mental cruelty on each other for a period of three years and that they have mutually agreed that the marriage should be dissolved. In the given situation, which of the following statements is correct?

(A) The district court shall pass a decree of divorce within six months after the presentation of petition for divorce by mutual consent.

(B) The district court shall pass a decree of divorce after eighteen months of the presentation of petition for divorce by mutual consent.

(C) The district court shall not immediately pass a decree of divorce by mutual consent.

(D) The district court shall immediately pass a decree of divorce by mutual consent if it is satisfied that the marriage was valid.

87. Ms. A, a Hindu female and Mr. B, a Christian male, got married as per the law relating to special marriages in January 2018. On February 15, 2021, Ms. A presented a petition for divorce by mutual consent before the district court on the ground that Ms. A and Mr. B have been living separately for a period of one year because Mr. B has been in an adulterous relationship with Ms. X, a Christian female. In the given situation, which of the following statements is correct?

(A) The district court shall pass a decree of divorce by mutual consent six months after the date of presentation of petition for divorce.

(B) The district court shall pass a decree of divorce by mutual consent eighteen months after the date of presentation of petition for divorce.

(C) The district court shall not pass a decree of divorce by mutual consent as the

requirements for grant of divorce are not fulfilled.

(D) The district court shall not pass a decree of divorce by mutual consent as Ms. A and Mr. B have not been living separately for more than one year.

88. Ms. A, a Hindu female and Mr. B, a Christian male, got married as per the law relating to special marriages in January 2018. On April10, 2021, Ms. A and Mr. B jointly presented a petition for divorce by mutual consent before the district court on the ground that they have been living separately for a period of three years and that they have mutually agreed that the marriage should be dissolved. In the given situation, which of the following statements is correct?

(A) The district court shall pass a decree of divorce by mutual consent after all other legal requirements are fulfilled.

(B) The district court shall pass a decree of divorce by mutual consent immediately because Ms. A and Mr. B have been living separately for more than one year.

(C) The district court shall not pass a decree of divorce by mutual consent because the marriage between Ms. A and Mr. B is not valid.

(D) The district court shall not pass a decree of divorce by mutual consent because neither party is at fault in the marriage.

89. Ms. A, a Hindu female and Mr. B, a Christian male, got married as per the provisions of the law relating to special marriages in January 2018. On June16, 2021, Ms. A and Mr. B jointly presented a petition for divorce by mutual consent before the district court on the ground that they have been living separately for a period of three years and that they have mutually agreed that the marriage should be dissolved. During the presentation of the petition, while Ms. A was present physically in the district court, Mr. B joined via video-conferencing. In the given situation, which of the following statements is correct?

(A) The district court may pass a decree of divorce by mutual consent six months after the date of presentation of petition for divorce.

(B) The district court shall pass a decree of divorce by mutual consent after all other legal requirements are fulfilled.

(C) The district court shall not pass a decree of divorce by mutual consent.

(D) Both (A) and (B).

90. Ms. A, a Hindu female and Mr. B, a Christian male, got married as per the law relating to special marriages in January 2018. On March15, 2021, Ms. A and Mr. B jointly presented a petition for divorce before the district court on the ground that they have been living separately for a period of three years and that they have mutually agreed that the marriage should be dissolved. Six months later, the district court, after hearing the parties and making inquiry, found that the marriage had been solemnized under the law relating to special marriages, and that the consent of Ms. A for the presentation of petition of divorce was obtained by fraud. In the given situation, which of the following statements is correct?

(A) The district court shall pass a decree of divorce by mutual consent because the legal requirements are fulfilled.

(B) The district court shall pass a decree of divorce because the marriage had been solemnized under the law relating to special marriages.

(C) The district court shall not pass a decree of divorce because there was no mutual consent between parties.

(D) The district court shall not pass a decree of divorce because Ms. A has not been punished for fraud.

XIX. There are two principal theories on the relationship between international law and domestic law- Monism and Dualism. The monistic theory maintains that the subjects of two systems of law, i.e., international law and municipal law are essentially one. The monistic theory asserts that international law and municipal law are fundamentally the same in nature, and arise from the same science of law, and are manifestations of a single conception of law. The followers of this theory view international law and municipal law as part of a universal body of legal rules binding all human beings, collectively or singly. In a monist system, international law does not need to be incorporated into domestic law because international law immediately becomes incorporated in domestic legal system upon ratification of an international treaty. According to this theory, domestic law is subordinate to international law. The Statute of the International Criminal Court, therefore, can be directly applied and adjudicated in national courts according to the monistic theory. According to dualism theory, international law and municipal law represent two entirely distinct legal systems, i.e., international has an intrinsically different character from that of municipal law. International law is not directly applicable in the domestic system under dualism. First, international law must be translated into State legislation before the domestic courts can apply it. For example, under dualism, ratification of the Statute of the International Criminal Court is not enough-it must be implemented through State legislation into the domestic system. Most states and courts presumptively view national and international legal systems as discrete entities and routinely discuss in dualist fashion incorporation of rules from one system to the other.

91. In light of the given passage, which of the following statements is correct?

(A) Monism and Dualism are similar approaches to adopt international law into domestic law.

(B) Dualism postulates the homogeneousness of domestic law and international law.

(C) Monism and Dualism are different approaches to understand how domestic law impacts international law.

(D) Monism postulates the homogeneousness of international law and domestic law.

92. ‘X’ is a developing country. ‘X’ ratified the United Nations Framework Convention on Climate Change in 1995, and incorporated the provisions of the said convention in its domestic legislation addressing climate change in 1996. However, ‘X’ has been widely criticized in the international community for its failure in meeting the obligations under the said convention. ‘Y’ is a developed country. ‘Y’ ratified the United Nations Framework Convention on Climate Change in 1995, and has not incorporated the provisions of the said convention in its domestic legislation till date. ‘Y’ has been appreciated by the international community for its success in meeting the obligations under the said convention. In the given situation, which of the following statements is correct?

(A) ‘X’ is a monist State and ‘Y’ is a dualist State.

(B) ‘X’ is a dualist State and ‘Y’ is a monist State.

(C) ‘X’ and ‘Y’ are both monist States.

(D) ‘X’ and ‘Y’ are both dualist States.

93. ‘D’, a dualist State, has signed and ratified the Agreement on Trade-related Aspects of Intellectual Property Rights (TRIPS Agreement), an international agreement administered by the World Trade Organization (WTO). If ‘D’ is compelled to fulfill its international obligations under the TRIPS Agreement, which of the following statements is correct?

(A) ‘D’ may adopt the provisions of the TRIPS Agreement without enacting a new domestic legislation or amending an existing legislation.

(B) ‘D’ may not incorporate the provisions of the TRIPS Agreement into a new domestic legislation.

(C) ‘D’ must incorporate the provisions of the TRIPS Agreement into an existing domestic legislation or in a new domestic legislation.

(D) ‘D’ may not incorporate the provisions of the TRIPS Agreement into an existing domestic legislation.

94. Which of the following statements is incorrect?

(A) According to monism, the nature of domestic law and international law is the same and domestic law is subordinate to international law.

(B) According to monism, ratified international conventions automatically become a part of domestic law and domestic law is subordinate to international law.

(C) According to dualism, ratified international conventions automatically become a part of domestic law and domestic law is subordinate to international law.

(D) According to dualism, the nature of domestic law and international law is different and domestic law is not subordinate to international law.

95. The country ‘X’ has ratified an International Convention which requires each State Party to enact laws defining and punishing bribery, i.e., the act of offering bribes to Government officials. The Convention has neither defined bribery, nor prescribed a punishment for the same, so that each State Party may define the offence of bribery differently in their respective domestic legislations. By 2022, ‘X’ has not enacted any law defining and punishing the offence of bribery. In November 2021, Mr. A was being prosecuted by a domestic criminal court in ‘X’ for allegedly offering a bribe to a Government official. In the given situation, which of the following statements is correct?

(A) If ‘X’ is a monist State, Mr. A can be punished for committing an offence under the Convention.

(B) If ‘X’ is a dualist State, Mr. A can be punished for committing an offence under the Convention.

(C) If ‘X’ is a dualist State, Mr. A cannot be punished for committing an offence under the Convention.

(D) Mr. A cannot be punished for committing an offence under the Convention irrespective of whether ‘X’ is a monist or a dualist State.

XX. The United Nations Commission on Environment and Development defines ‘sustainable development’ as follows: “Sustainable development is the development that meets the needs of the present without compromising the ability of future generations to meet their own needs.” Sustainable development clearly postulates an anthropocentric bias, least concerned with the rights of other species which live on this earth. Anthropocentrism is always human-interest focused thinking that considers non-humans as having only instrumental value to humans, in other words, humans take precedence and human responsibilities towards non-human are based on benefits to humans. Eco-centrism is nature-centred, where humans are part of nature and non-humans have intrinsic value. In other words, human interest does not take automatic precedence and humans have obligations towards non-humans independently of human interest. Eco-centrism is, therefore, life-centred, nature-centred where nature includes both humans and non-humans. The Constitution of India protects not only human rights but also casts an obligation on human beings to protect and preserve a specie from becoming extinct. Conservation and protection of environment is an inseparable part of the fundamental right to life. According to the doctrine of ‘public trust’ recognized under the Constitution of India, certain common properties such as rivers, seashores, forests and the air are held by the Government in trusteeship for the free and unimpeded use of the general public. The resources like air, sea, waters and the forests have such a great importance to the people as a whole, that it would be totally unjustified to make them a subject of private ownership. The State, as a custodian of the natural resources, has a duty to maintain them not merely for the benefit of the public, but for the best interest of flora and fauna, wildlife and so on.

96. Ms. G, a student of environmental science, has cultivated a butterfly garden which provides a favourable habitat for butterflies. Ms. G has cultivated the butterfly garden so that she could observe and study the different stages of development of butterflies such as egg, larvae, pupae, and adult. In the given situation, which of the following statements is correct?

(A) Ms. G’s approach to cultivation of butterfly garden is anthropocentric because it concerns the furtherance of her academic interest.

(B) Ms. G’s approach to cultivation of butterfly garden is anthropocentric because it concerns the conservation of environment.

(C) Ms. G’s approach to cultivation of butterfly garden is eco-centric because it concerns the provision a favourable habitat to the butterflies.

(D) Ms. G’s approach to cultivation of butterfly garden is eco-centric because it concerns the understanding of the different stages of development of butterflies.

97. Which of the following statements is correct?

(A) Anthropocentrism and eco-centrism are different approaches to achieving sustainable development.

(B) Anthropocentrism focuses on the promotion of non-human interests.

(C) Eco-centrism is concerned with the promotion of both human and non-human interests.

(D) Anthropocentrism and eco-centrism are different approaches to protection of environment and sustainable development.

98. Which of the following is the basis for the difference between anthropocentrism and eco-centrism?

(A) The inherent value placed on humans and non-humans.

(B) The inherent value placed on living things and non-living things.

(C) The relationship between human society and environment.

(D) The relationship between non-humans and environment.

99. Which of the following statements is correct regarding fundamental right to life under the Constitution of India?

(A) It creates a corresponding duty on human beings to protect and preserve non-humans.

(B) It creates a corresponding duty on non-humans to protect the right to life of human beings.

(C) It is inclusive of the right of human beings to utilize non-human resources to the best of their advantage.

(D) It is inclusive of the right of non-humans to utilize human resources to the best of their advantage.

100. Which of the following statements is incorrect according to the doctrine of ‘public trust’?

(A) Private ownership of forests is unwarranted.

(B) Forests are held by the State in a fiduciary capacity.

(C) State is obligated to maintain forests for their economic value.

(D) State is obligated to maintain forests in the interest of humans and non-humans.

XXI. When parties to a contract are under a ‘mistake’ regarding an important fact related to such contract, it may affect the contract in two ways. It may, firstly, defeat the consent altogether that the parties are supposed to have given, that is to say, the consent is unreal. Two or more persons are said to consent when they agree upon the same thing in the same sense. Secondly, the mistake may mislead the parties as to the purpose which they had contemplated. Where the mistake does not defeat consent, but only misleads the parties, i.e., where both parties to an agreement are under a mistake as to a matter of fact essential to the agreement, the agreement is void. However, if the mistake is concerning an erroneous opinion regarding value of the subject-matter of the agreement, it is not a mistake as to a matter of fact. Thus, agreement is void when: (1) both the parties to an agreement are mistaken, (2) their mistake is as to a matter of fact, and (3) the fact about which they are mistaken is essential to the agreement. Further, it is pertinent to note that a mistake, in order to invalidate a contract, should be a mistake of fact and not a mistake of law. Furthermore, where only one party to the contract is under mistake of fact, and the other party is not, the contract is not voidable merely for such reason.

101. Mr. A entered into an agreement to sell his bicycle which had been kept unused in his attic for a year, to Mr. B, at an agreed price. However, neither party was aware that at the time of entering into the agreement, the bicycle had already been destroyed by a fire in the attic. In the given situation, which of the following statements is true?

(A) The agreement is void as both parties were under a mistake as to a matter of fact essential to the agreement.

(B) The agreement is void as both parties were under a mistake as to a law in force in India.

(C) The agreement is not voidable as only one of the parties was under a mistake as to a matter of fact.

(D) The agreement is not voidable as the promise made under the agreement had not been performed.

102. Ms. X and Ms. Y entered into a contract of sale of an article, while reeling under the erroneous belief that the sale of the article, which was the subject-matter of the agreement, was permitted by the law in force in India. In the given situation, which of the following statements is true?

(A) The contract is valid.

(B) The contract is voidable at the option of Ms. X.

(C) The contract is voidable at the option of Ms. Y.

(D) The contract is not voidable.

103. Mr. J entered into an agreement with Mr. K for the sale of Mr. J’s ‘club’. At the time of entering into the agreement, while Mr. J believed that he was agreeing to sell his golf club, Mr. K believed that he was agreeing to buy a clubhouse owned by Mr. J. The agreement is void because:

(A) Mistake of fact defeated the consent of the parties.

(B) Mistake of fact misled the parties as to the purpose of the contract.

(C) Mistake of fact was regarding the identity of parties.

(D) Both (A) and (B)

104. Mr. D appointed Mr. K to manage the cultivation of his land as he was unable to manage it himself due to his advanced age. Mr. K agreed to manage the cultivation of Mr. D’s land if he granted Mr. K a lease of the said land. Mr. D agreed to the same and signed a deed which was, unknown to both parties, a gift deed of the land and not a lease deed. In the given situation, which of the following statements is true?

(A) Mr. D and Mr. K were reeling under a mistake as to a matter of fact essential to the agreement.

(B) Mr. D and Mr. K were reeling under a mistake as to a matter of law essential to the agreement.

(C) Mr. D was reeling under a mistake as to a matter of fact essential to the agreement.

(D) Mr. D was reeling under a mistake as to a matter of law essential to the agreement.

105. Ms. X and Ms. Y entered into a contract of sale of an article which was agreed to be shipped by Ms. X in a ship named ‘The Cruiser’ and delivered to Ms. Y on an agreed date. Mr. X shipped the said article by a different ship named ‘The Mariner’, without informing Ms. Y and the article was delivered to Ms. Y on the agreed date. In the given situation, which of the following statements is true?

(A) The agreement is void as both parties were under a mistake as to a matter of fact essential to the agreement.

(B) The agreement is void as both parties were under a mistake as to a law in force in India.

(C) The agreement is valid as both parties were under a mistake as to a matter of fact not essential to the agreement.

(D) The agreement is valid as both parties were under a mistake as to a matter of fact essential to the agreement.

Logical Reasoning

XXII. Students decide to attend college for several reasons. These reasons include career opportunities and financial stability, intellectual growth, a time for self-discovery, norms, obligations, and social opportunities. Outside demands in society, such as technology changes, and increased educational demands also drive the need for more students to attend college. The students then spend the next few years trying to discover a path and find their way so they can become successful. The transition to college presents students with many new challenges, including increased academic demands, less time with family members, interpersonal problems with roommates and romantic interests, and financial stress. Competitive academic work and uncertainty about future employment and professional career were also noted as sources of stress. The transition to college represents a process characterized by change, ambiguity, and adjustment across all of life’s domains. The transition towards independence and self-sufficiency has been characterized as ‘stress-arousing’ and ‘anxiety-provoking’ by many college students. Failure to accomplish and develop these characteristics of development and maintain independence may result in life dissatisfaction. Emerging adulthood has also been noted to augment college students’ vulnerability to stress. Many students experience their first symptoms of depression and anxiety during this time, but a growing problem is that college campuses do not have enough resources to help all of these students. It has been noted that 75% to 80% of college students are moderately stressed and 10% to 12% are severely stressed.

106. What according to you is the objective of the study of the present paragraph?

(A) To map the various stages of pressure points of adulthood in the process of education.

(B) To narrate the anti-family agenda in the current education system.

(C) To pinpoint the obstacles targeted against meritorious students.

(D) All of the above

107. Which factors as per the author cause more stress amongst college students?

(A) Pressure from parents and society towards greater educational needs and increased competitive academic work.

(B) Failure to develop successful romantic interests, financial constraints and interpersonal issues with room-mates.

(C) Failure to adapt to the transition to college life and to adjust various life domains in tune with needs and requirements of college life.

(D) Inability to manage time constraints and the uncertainty pertaining to their future.

108. Which of the following fall closest to the underlying assumption in the present study?

(A) Problem-solving ability amongst college students is negatively associated with symptoms of depression and anxiety.

(B) Students lean towards unhealthy coping skills in order to try to lower the stress that they experience.

(C) Romantic interest is an anti-dote for stress amongst the students in the colleges.

(D) Stress is subjective for each student.

109. Suggest a suitable title for the paragraph from amongst the given titles:

(A) Triumph and Turbulence of College Education System

(B) Negative impact of College Education System

(C) Negligence of Stress Management by parents

(D) Unemployment and Mental Instability

110. With reference to the above paragraph, which of the following offers the most plausible solutions as a coping up mechanism for college students?

(A) Individual students should approach counsellors for coping up with stress.

(B) Keeping in view that large number of students are experiencing stress, colleges must take steps reduce course curriculum and peer pressures.

(C) College authorities shall provide access to counselling and every student experiencing stress must engage in some form of coping mechanism to alleviate stress.

(D) The students must learn to differentiate between short term and long-term stress.

XXIII. Under the COVID-19 outbreak, universities and schools around the world had suspended face-to-face classes to prevent the rapid spread of the virus among students and staff. This sudden disruption to face-to-face education reshaped pedagogical practices and led to the rapid adoption of online teaching among universities. Subsequently, academics working at universities, at the frontline of those changes, faced enormous levels of pressure and disturbance to their professional roles and practices. For those without sufficient knowledge or experience for effective online teaching, this sudden transition was particularly challenging. In normal circumstances, designing an online course follows a systematic instructional design process with careful consideration of the unique characteristics of target learner groups and the chosen instructional medium. During the rapid adoption of online teaching in response to COVID-19, however, systematic instructional design procedures and team-based support for course development and preparation were unavailable. Instead, individual academics were given the challenge alone to teach online with a limited level of support and guidance from their school or university – the task was even more difficult in this situation where they were remotely working from home.

111. The objective behind the information furnished in the passage is:

(A) To examine the experiences faced by the teachers because of the sudden transition from offline to online mode of teaching due to outbreak of COVID-19.

(B) To identify the differences between the online and offline mode of teaching.

(C) To reveal the side-effects of COVID-19.

(D) To understand the need to be able to cope up with crisis like situations even in the educational sector.

112. Based on the ideas presented in the paragraph, it will not be possible to draw out useful recommendations for situations like the pandemic, unless :

(A) The factors required to contribute to quality education by online and offline modes are examined.

(B) Knowledge about the infrastructural availability in the schools or universities is crucial.

(C) The faculty is given adequate training and experience in providing online education is taken into consideration.

(D) A detailed analysis of the comparative performance in the online and offline modes is done.

113. What can be most conveniently inferred from the given paragraph?

(A) Whether online or face-to-face, university teaching activity is a genuinely complex task that involves multiple elements of interlinked activity systems.

(B) It has been more challenging for both individual academics and institutions to quickly adopt to online teaching during the COVID-19 Pandemic.

(C) The object of the online teaching activity systems created a fundamental contradiction with the object of the previous teaching activity systems.

(D) All the above.

114. What suitable policy decision should be devised by the administrators of the schools and the universities, in the light of the facts presented in the paragraph?

(A) There must be an insight into the complexity of online teaching and need to work for the capacity building of the teachers during such extra ordinary times and there is a greater need to create a teacher community and foster collaborative teaching relationships among the members, even if it takes time.

(B) The faculty members must be oriented towards the lasting changes brought about to their roles and identities in teaching.

(C) It is time to develop a comprehensive understanding of the challenges experienced by individual academics and the changes created by those academics.

(D) There is a need to develop infrastructure in schools and universities.

115. Which of the following points most closely supports the fact that the present education system lacks the structure to sustain effective teaching during and after the periods of lockdowns?

(A) There are inadequate applications and platforms for effective online teaching.

(B) Shift from offline to online was faced with resistance.

(C) The academia’s long-established roles and identities have been completely altered by the pandemic.

(D) The students are interested in online examinations and schools and universities are finding it difficult to shift to offline examination mode.

XXIV. Biodiversity is being lost at a rate not seen since the last mass extinction. But the United Nations decade-old plan to slow down and eventually stop the decline of species and ecosystems by 2020 has failed. Most of the plan’s 20 targets – known as the Aichi Biodiversity Targets – have not been met. The Aichi targets are part of an international agreement called the UN Convention on Biological Diversity, and member states are now finalizing replacements for them. Currently referred to as the post-2020 Global Biodiversity Framework (GBF), its draft was published in July 2021. It aims to slow down the rate of biodiversity loss by 2030. And by 2050, biodiversity will be “valued, conserved, restored and wisely used, maintaining ecosystem services, sustaining a healthy planet and delivering benefits essential for all people”. The GBF is a comprehensive plan. But success will require systemic change across public policy. That is both a strength and a weakness. If systemic change can be implemented, it will lead to real change. But if it cannot, there’s no plan B. This has led some researchers to argue that one target or number should be prioritized and defined in a way that is clear to the public and to policy makers. It would be biodiversity’s equivalent of the 2°C climate target.

116. As per the passage, which of the following is a challenge for implementation of the post-2020 Global Biodiversity Framework (GBF)?

(A) Unfulfillment of the pre-2020 global biodiversity targets

(B) Clarity of action plan for the society and government

(C) Threat of mass extinction

(D) Failure of plan to save ecosystems

117. According to the passage, why do some researchers advocate that one biodiversity target be prioritized?

(A) Systemic policy change is difficult to implement

(B) Post-2020 Global Biodiversity Framework (GBF) is comprehensive

(C) The 2 °C climate target needs to be prioritized

(D) Biodiversity needs to be valued, conserved, restored and wisely used

118. Which of the following is correct expression of the author’s opinion as stated in passage?

(A) Implementation of the post-2020 Global Biodiversity Framework (GBF) allows no middle ground for success or failure.

(B) It is high time that countries re-evaluate the progress in achieving biodiversity targets.

(C) Biodiversity conservation should be prioritized over climate change at the global level.

(D) The post-2020 Global Biodiversity Framework (GBF) is the best way to prevent mass extinction.

119. Which of the following is the central theme of the above passage?

(A) The reason for failure of biodiversity conservation efforts at global level

(B) The inter-relationship between biodiversity conservation and climate change

(C) The future of biodiversity conservation efforts at global level

(D) The role of United Nations in biodiversity conservation at global level

120. Which of the following can be inferred from the above passage?

(A) The holistic nature of the post-2020 Global Biodiversity Framework (GBF) is a boon.

(B) The holistic nature of the post-2020 Global Biodiversity Framework (GBF) is a bane.

(C) The holistic nature of the post-2020 Global Biodiversity Framework (GBF) can be a boon or a bane.

(D) The post-2020 Global Biodiversity Framework (GBF) includes the 2°C climate target.

XXV. An unintended and unjust consequence of the Protection of Children from Sexual Offences Act, 2012 is its widespread persecution of teenage lovers. This law raised the age of consent from 16 to 18 years, while defining persons below 18 years as children. Consequently, when two 16-year-olds are romantically and sexually involved, but the girl’s family doesn’t approve the affair and files a police complaint, her consent has zero legal validity. And the consensual relationship morphs into a case of statutory rape. The Allahabad High Court has indicated how its “conscience” is concerned by such severe POCSO provisions being drawn by teenage lovers simply on the basis of family disapproval. The High Courts of Delhi, Madras and others have made similar observations in recent years and also pointed to amendments to the law that can help reduce its injustices. One suggestion that has gathered broad support is to push back both the cut-off for childhood and the age of consent to 16 years. Given that the NCRB data shows around half of POCSO cases falling in the 16-18 years age group, such an amendment is overdue. Minimizing the prosecution of consensual romances would also leave a logjammed system with more space to pursue actual sexual assault cases. The broader goal here is respecting the rights of adolescents and young adults. Their romantic and sexual autonomy needs greatly increased recognition in India.

121. “Minimising the prosecution of consensual romances would also leave a logjammed system with more space to pursue actual sexual assault cases.”

In the context of the statement, which of the following strengthens the author’s opinion?

(A) There are many unreported sexual assault cases.

(B) Speedy prosecution of sexual assault cases is desirable.

(C) Consensual romance, in some cases, can amount to sexual assault.

(D) Sexual assault and rape are different.

122. What has the author conveyed regarding the prosecution of statutory rape in India?

(A) Statutory rape does not violate the romantic and sexual autonomy of young adults.

(B) Statutory rape must be met with stricter punishment.

(C) Statutory rape must be abolished.

(D) Statutory rape is a relic of Victorian morality.

123. As per the above passage, which of the following does not correctly represent the author’s view regarding the widespread persecution of teenage lovers under the POCSO Act?

(A) Teenagers have the right to love as much as adults.

(B) Police complaints of teenage lovers may lead to their harassment.

(C) Consent of minor girls do not have legal validity.

(D) Consent of minor girls have legal validity.

124. In the above passage, which of the following has concerned the “conscience” of the Allahabad High Court?

(A) Carelessness of teenage lovers.

(B) Threat to the safety of teenage lovers.

(C) Impact on mental health of teenage lovers.

(D) Harassment of teenage lovers by their families.

125. As per the above passage, which of the following is a major challenge in implementation of the POCSO Act?

(A) Speedy prosecution of cases.

(B) Respecting the rights of adolescents and young adults.

(C) Counselling of adolescents and young adults.

(D) Imposition of stringent punishment.

XXVI. A Madras High Court Judge’s suggestion to amend the Constitution of India mandating that every citizen also has a duty to laugh comes as a whiff of fresh air – something the country has been gasping for, of late. Justice GR Swaminathan of the Madurai Bench has a remarkably refined sense of humour, but in quashing an FIR against a man arrested for an innocuous social media post, his insightful observations only highlight the idiocy and absurdity that surround the growth and normalisation of the offence-taking tribe. Written from the perspective of cartoonists and satirists, the judgment draws attention to how what ought to be a reasonable understanding of a situation is increasingly being influenced by impulses that border on the irrational and amount to an abuse of the legal process. The petitioner tried tongue-in-cheek wordplay while captioning photographs after a sight-seeing trip with family : ‘Trip to Sirumalai for shooting practice’. For the police, it appeared as a threat to wage war, though the Judicial Magistrate refused remand. ‘Laugh at what?’ is a serious question, the Judge said, using the ‘holy cow’ as a metaphor, which varies from person to person, region to region. Being funny is one thing, the Judge righty states, and poking fun at another is different altogether. Those who have been at the receiving end for their attempt at humour can draw strength from the ruling, but then, a creative process facing combative opposition because of its very nature is anything but funny.

126. What is the central idea in the passage as conveyed by the author?

(A) People need to be sensitive towards others’ sensibilities.

(B) Humour is often used as a garb to offend others.

(C) There is an unwelcome decrease in people’s sense of humour.

(D) Judiciary should use humour to make judgments understandable to laypersons.

127. According to the given passage, which of the following statements is true?

(A) Social media often popularised insensitive and offensive posts.

(B) It should be a right of every person to poke fun at others.

(C) Creative expressions are bound to be offensive to some persons.

(D) Every humorous expression should be understood reasonably and rationally.

128. According to the given passage, which of the following statements is not true?

(A) Each expression should be understood according to its context.

(B) The word ‘shooting’ used in a sentence is indicative of waging war.

(C) Legal process can be abused if the authorities act on their irrational impulses.

(D) The expression ‘holy cow’ bears different meanings for different people.

129. As per the passage, which of the following approaches can reduce the increasing ‘idiocy and absurdity’ in responding to expressions made in jest?

(A) Apologising upon hurting another person’s sentiments.

(B) Avoiding the use of controversial words and expressions.

(C) Using humour as a means to mitigate conflict.

(D) Understanding the difference between being funny and poking fun at another person.

130. “Those who have been at the receiving end for their attempt at humour can draw strength from the ruling, but then, a creative process facing combative opposition because of its very nature is anything but funny.”

Which of the following conclusions can be drawn from the above statement?

(A) Creativity and conflict go hand-in-hand.

(B) Creative freedom should not be curbed unreasonably.

(C) Creative expressions are strengthened due to challenges faced by their authors.

(D) Creativity often leads to conflict.

XXVII. Two recent developments have brought India’s reliance on fossil fuel into sharp focus. The Russia-Ukraine conflict and the consequent surge in crude oil prices roiled the economy. Separately, the most recent IPCC report on climate highlighted the energy sector’s large contribution to global warming. Both these developments need to be located in the context of India’s pledge to get to net zero carbon emissions by 2070. Meeting this pledge requires an overhaul of both the logistics and electricity sectors to reduce reliance on fossil fuels. Transitioning to renewables in energy is an important part of the solution. Within renewables, solar energy has been lavished with policy support. However, it won’t be enough to meet the targets. Anil Kakodkar, former chairman of Atomic Energy Commission, had written that India can’t meet its net-zero commitment without nuclear power. He’s right. It’s an area where India was off to an early start, developed relatively high indigenous capabilities in relation to other sectors, but subsequently let the ball drop. Today, nuclear power contributes a mere 3% of the total electricity generated, and has a capacity of 6780 MW. After the early euphoria of the India-United States civil nuclear deal, progress has been disappointing. The deal did open the pathway to a stable supply of uranium ore from Kazakhstan and Canada. However, the design of the subsequent bill on civil liability for nuclear damage killed the prospect of participation of Western firms. India’s main partner today is Russia, which side stepped the bill through inter-government agreements.

131. What is the central idea in the passage as conveyed by the author?

(A) India needs to increase use of nuclear power.

(B) India needs to increase production of fossil fuels.

(C) India needs to enter into multilateral agreements addressing use of nuclear power.

(D) Nuclear energy is a renewable source of power.

132. According to the author, which of the following measures will not help India achieve its pledge of net zero carbon emissions by 2070?

(A) Logistical changes

(B) Changes in electricity sector

(C) Reduction in use of solar power

(D) Increase in use of nuclear power

133. According to the author, which of the following is not the effect of India’s reliance on fossil fuels?

(A) Global warming

(B) Increase in crude oil prices

(C) Relations with Middle East

(D) Less reliance on renewable sources of power

134. According to the author, which of the following is the effect of the India-United States civil nuclear deal?

(A) Export-Import target with United States

(B) Removal of bottlenecks for self-reliance in power generation

(C) Nuclear Defence Pact with Kazakhstan

(D) Self-reliance in Solar Power

135. According to the author, Western firms lost the opportunity of doing business in the nuclear production in India because:

(A) They had to pay hefty penalties for delay in supply

(B) They do not find nuclear power profitable

(C) They do not agree with India’s place of nuclear plants

(D) They failed to circumvent internal laws by other bilateral instruments

Quantitative Techniques

XXVIII. As per a survey conducted in a college out of total students enrolled i.e., 3,000 in 2020-21, 1,700 were girls and 1,300 were boys. Data regarding students opting for various streams viz., Non-Medical, Medical, Commerce, Arts and Fine Arts showed that 25% of the enrolled students opted non-medical and the percentage of girls in Non-Medical was 30% of the total number of girls; 15% of the total students opted for Medical and the percentage of girls who opted Medical was 18% of the total number of girls; 25% of the total students opted Arts but the percentage of girls who opted for Arts was 15% of the total number of girls; 16% of the total students opted Commerce and the percentage of girls who opted Commerce was 17% of the total number of girls, and; 19% of the total students opted Fine Arts and the percentage of girls who opted Fine Arts was 20% of the total number of girls.

136. How many girls have opted Non-Medical?

(A) 440

(B) 365

(C) 530

(D) 510

137. Girls have outnumbered boys in Fine Arts. How many girls in Fine Arts are more than the boys, as a percentage of total number of boys in Fine Arts?

(A) 49.62%

(B) 47.82%

(C) 51.23%

(D) 50.89%

138. Which of the following courses have been opted by maximum number of boys?

(A) Non-Medical

(B) Arts

(C) Fine Arts

(D) Commerce

139. Which of the following courses have been opted by minimum number of boys?

(A) Medical

(B) Fine Arts

(C) Commerce

(D) Non-Medical

140. What is the ratio among boys and girls for Non-Medical?

(A) 3 : 17

(B) 17 : 8

(C) 8 : 17

(D) 17 : 3

XXIX. An Indian company, having its registered office at Gurugram, is engaged in manufacturing of consumer goods at Noida. The goods manufactured by the company are sold in Indian market and exported to Europe. Company produces five products namely ‘P’,‘Q’,‘R’,‘S’ and ‘T’. Total production of the company for the financial year 2021-22 is 3,000 tonnes and the turnover of the company is ` 50 million. An analysis of the production and net revenue generation shows that production of product ‘P’ is 21% of the total production and 18% of the turnover is attributable to product ‘P’; production of ‘Q’ is 16% of the total production and 17% of the turnover is attributable to ‘Q’; ‘R’ accounts for 18% of the total production and 20% of the turnover; ‘S’ accounts for 20% of the total production and 25% of the turnover, and; ‘T’ accounts for 25% of total production and 20% of turnover.

141. What is the percentage of profit earned from sale of ‘R’, if the expenditure incurred on production of ‘R’ is ` 15,000/- per tonne?

(A) 20%

(B) 23.46%

(C) 26.55%

(D) 25%

142. Which product has the highest selling price per tonne?

(A) Q

(B) R

(C) S

(D) T

143. How much loss is incurred to company, if the expenditure on production of ‘T’ was `20,000 per tonne?

(A) 5 million

(B) 10 million

(C) 4 million

(D) 7 million

144. What percentage of turnover of ‘R’ has to turnover of ‘T’?

(A) 100%

(B) 75%

(C) 50%

(D) 60%

145. What is the average selling price per tonne of all products taken together?

(A) ` 17,488

(B) ` 17,667

(C) ` 18,667

(D) ` 16,667

XXX. In an organization, the total number of employees working in various Departments viz. IT, Marketing, Purchase, HR, Accounts and Production are 4,500. The information regarding department wise percentage of employees was collected and also record about gender ratio of employees was prepared. 18 percent of total number of employees work in IT department and ratio of males to females in IT department is 2 : 1. In Marketing, ratio of males to females is 2 : 3 and number of employees engaged in marketing is 20% of the total employment. 12% of the total numbers of workers are running the HR department and the ratio of males to females in this department is 5 : 1. The fraction of male to females in production department is 3 : 2 and total number of persons employed in this department is 15% of the total workforce. The number of persons occupied in purchase and accounts department is 24% and 11% respectively of the total number of workers. Gender Ratio (Ratio of males to females) in Purchase department is 1 : 1 and in Accounts is 1 : 2.

146. How many females are employed in Purchase department?

(A) 450

(B) 540

(C) 495

(D) 595

147. How many employees are working in IT and Accounts departments together?

(A) 1,702

(B) 1,646

(C) 1,766

(D) 1,305

148. What is the ratio of total number of males to total number of females working in all the departments put together?

(A) 63 : 41

(B) 19 : 27

(C) 41 : 34

(D) 34 : 41

149. Number of females in Marketing Department forms what percentage of the total number of employees in the organization?

(A) 8%

(B) 7%

(C) 12%

(D) 10%

150. What is the ratio of number of males in Marketing Department to the number of males in HR department?

(A) 4: 5

(B) 5: 4

(C) 7 : 3

(D) 6: 7

Click Here for Online Mock Tests and Solve Live.

Answer

1) B

2) C

3) D

4) C

5) A

6) D

7) C

8) B

9) B

10) A

11) D

12) A

13) B

14) D

15) C

16) C

17) D

18) B

19) A

20) D

21) C

22) D

23) B

24) C

25) C

26) B

27) D

28) A

29) A

30) D

31) B

32) C

33) D

34) C

35) A

36) D

37) C

38) C

39) A

40) D

41) D

42) A

43) D

44) C

45) B

46) C

47) A

48) B

49) A

50) D

51) B

52) C

53) C

54) D

55) A

56) D

57) B

58) A

59) C

60) D

61) A

61) C

63) D

64) A

65) B

66) A

67) B

68) C

69) A

70) D

71) B

72) B

73) C

74) C

75) A

76) B

77) B

78) A

79) D

80) D

81) C

82) D

83) B

84) D

85) A

86) C

87) C

88) A

89) D

90) C

91) D

92) B

93) C

94) C

95) D

96) A

97) C

98) A

99) A

100) C

101) A

102) D

103) D

104) A

105) C

106) A

107) C

108) A

109) A

110) C

111) A

112) C

113) D

114) A

115) C

116) B

117) A

118) A

119) C

120) C

121) B

122) A

123) D

124) D

125) C

126) C

127) D

128) B

129) D

130) B

131) A

132) C

133) C

134) B

135) D

136) D

137) B

138) B

139) A

140) C

141) B

142) C

143) A

144) A

145) D

146) B

147) D

148) C

149) C

150) A

Important Links

Law Library: Notes and Study Material for LLB, LLM, Judiciary, and Entrance Exams

Law Aspirants – Ultimate Test Prep Destination

Read More